由买买提看人间百态

boards

本页内容为未名空间相应帖子的节选和存档,一周内的贴子最多显示50字,超过一周显示500字 访问原贴
Mathematics版 - one problem about infinite products
相关主题
what's the genus of cos(sqrt(z))?问个数学分析的证明题,在线急等
[合集] one problem about infinite products求教两个正项级数的极限证明
help! Matrix question (interveiw question)请教一个级数公式的证明
这个series 是convergent 还是divergent?[求助]cos(a)+K*cos(b) 如何化简成一个乘积形式
再来Ahlfors书上一道习题O(log(n))分解只有两个质数因子的合数
这个级数收敛么?问一个基础的分布收敛问题,谢谢!
真心求教一道随机收敛的问题Parabolic equation收敛问题
convergence 和 validation看看这个,即收敛又发散的调和级数
相关话题的讨论汇总
话题: prod话题: sum话题: 收敛话题: sqrt话题: log
进入Mathematics版参与讨论
1 (共1页)
o**a
发帖数: 76
1
Ahlfors said that the convergence of \sum a_n is neither sufficient nor
necessary condition for the convergence of \prod (1+a_n).
(a_n are complex numbers)
I've worked out the example that \prod (1+a_n) converges but \sum a_n doesn't.
For example, a_n=-1/(n+1), \sum a_n -> -\infty, but \prod (1+a_n) -> 0
Actually, as long as for all a_n, -1 < a_n < 0, then \prod (1+a_n) is positive
and decreasing, so converges, but obviously \sum a_n may diverge.
My question is, can you think of a series {a_n} s
x******g
发帖数: 318
2
你那个"反例"给得不好,因为极限是负无穷大也算是有极限的.
实际上log(1+an)约等于an,所以\sum a_n=-00基本上和\prod (1+a_n)=0差不多的,不能算
是反例.
两个反例
1.
a_n=(-1)^n*(n+1)
\sum a_n存在,\prod (1+a_n)不存在
2.
a_n=-1 n=1
随意,但要保证\sum a_n不存在 n为其他值时
\sum a_n不存在,\prod (1+a_n)=0

t.
positive

【在 o**a 的大作中提到】
: Ahlfors said that the convergence of \sum a_n is neither sufficient nor
: necessary condition for the convergence of \prod (1+a_n).
: (a_n are complex numbers)
: I've worked out the example that \prod (1+a_n) converges but \sum a_n doesn't.
: For example, a_n=-1/(n+1), \sum a_n -> -\infty, but \prod (1+a_n) -> 0
: Actually, as long as for all a_n, -1 < a_n < 0, then \prod (1+a_n) is positive
: and decreasing, so converges, but obviously \sum a_n may diverge.
: My question is, can you think of a series {a_n} s

o**a
发帖数: 76
3
that makes some sense



【在 x******g 的大作中提到】
: 你那个"反例"给得不好,因为极限是负无穷大也算是有极限的.
: 实际上log(1+an)约等于an,所以\sum a_n=-00基本上和\prod (1+a_n)=0差不多的,不能算
: 是反例.
: 两个反例
: 1.
: a_n=(-1)^n*(n+1)
: \sum a_n存在,\prod (1+a_n)不存在
: 2.
: a_n=-1 n=1
: 随意,但要保证\sum a_n不存在 n为其他值时

o**a
发帖数: 76
4
It seems that
a_n = (-1)^(n+1) * i/\sqrt(n) gives the example for the second case that
\sum a_n converges but \prod (1+a_n) diverges
for \sum a_n goes to some imaginary number, but the module of \prod (1+a_n)
goes to infinity

t.
positive

【在 o**a 的大作中提到】
: Ahlfors said that the convergence of \sum a_n is neither sufficient nor
: necessary condition for the convergence of \prod (1+a_n).
: (a_n are complex numbers)
: I've worked out the example that \prod (1+a_n) converges but \sum a_n doesn't.
: For example, a_n=-1/(n+1), \sum a_n -> -\infty, but \prod (1+a_n) -> 0
: Actually, as long as for all a_n, -1 < a_n < 0, then \prod (1+a_n) is positive
: and decreasing, so converges, but obviously \sum a_n may diverge.
: My question is, can you think of a series {a_n} s

o**a
发帖数: 76
5


this is not correct

【在 x******g 的大作中提到】
: 你那个"反例"给得不好,因为极限是负无穷大也算是有极限的.
: 实际上log(1+an)约等于an,所以\sum a_n=-00基本上和\prod (1+a_n)=0差不多的,不能算
: 是反例.
: 两个反例
: 1.
: a_n=(-1)^n*(n+1)
: \sum a_n存在,\prod (1+a_n)不存在
: 2.
: a_n=-1 n=1
: 随意,但要保证\sum a_n不存在 n为其他值时

x******g
发帖数: 318
6
写错了,等一下我要修改

【在 o**a 的大作中提到】
:
: 算
: this is not correct

x******g
发帖数: 318
7
我要去打乒乓球了,等会在想那个反例

【在 o**a 的大作中提到】
:
: 算
: this is not correct

o**a
发帖数: 76
8
so healthy :-)

【在 x******g 的大作中提到】
: 我要去打乒乓球了,等会在想那个反例
o**a
发帖数: 76
9


this is not a counter example
by definition, \prod (1+a_n) converges if and only if
finite number of a_n are -1, and the partial products formed by the
nonvanishing factors tend to a finite limit which is different from 0
所以我刚才给的反例是错的, partial products趋于0,这infinite products不能说是
converge的,原因大概就是你上面给出的:那相当于\sum a_n=-00

【在 x******g 的大作中提到】
: 你那个"反例"给得不好,因为极限是负无穷大也算是有极限的.
: 实际上log(1+an)约等于an,所以\sum a_n=-00基本上和\prod (1+a_n)=0差不多的,不能算
: 是反例.
: 两个反例
: 1.
: a_n=(-1)^n*(n+1)
: \sum a_n存在,\prod (1+a_n)不存在
: 2.
: a_n=-1 n=1
: 随意,但要保证\sum a_n不存在 n为其他值时

w******o
发帖数: 442
10
what is the definition of prod (1+a_n) and its convergent

【在 o**a 的大作中提到】
:
: 算
: this is not a counter example
: by definition, \prod (1+a_n) converges if and only if
: finite number of a_n are -1, and the partial products formed by the
: nonvanishing factors tend to a finite limit which is different from 0
: 所以我刚才给的反例是错的, partial products趋于0,这infinite products不能说是
: converge的,原因大概就是你上面给出的:那相当于\sum a_n=-00

相关主题
这个级数收敛么?问个数学分析的证明题,在线急等
真心求教一道随机收敛的问题求教两个正项级数的极限证明
convergence 和 validation请教一个级数公式的证明
进入Mathematics版参与讨论
o**a
发帖数: 76
11
Ahlfors书上对无穷乘积
p_1*p_2*...*p_n...=\prod p_n (*)
的定义是
"The infinite product (*) is said to converge if and only if at most a finite
number of the factors are 0, AND if the partial products formed by the
nonvanishing factors tend to a finite limit which is different from zero."
翻译成中文就是:无穷乘积 p_1 * p_2 * ... * p_n * ...收敛的定义是 p_i 中最多有
有限个是0,并且不是0的p_i构成的部分乘积收敛到一个有限的但不是0的数。
例如
(1) 0 * 1 * 2 * ...* n *..不收敛,因为去掉0后的部分乘积不收敛。
(2) (1-1/2) * (1-1/3) * ... * (1-1/n) * ...不收敛,因为部分乘积收敛到0
(2) (1-1/2^2) * (1-1/

【在 w******o 的大作中提到】
: what is the definition of prod (1+a_n) and its convergent
w******o
发帖数: 442
12
thank you very much

finite


【在 o**a 的大作中提到】
: Ahlfors书上对无穷乘积
: p_1*p_2*...*p_n...=\prod p_n (*)
: 的定义是
: "The infinite product (*) is said to converge if and only if at most a finite
: number of the factors are 0, AND if the partial products formed by the
: nonvanishing factors tend to a finite limit which is different from zero."
: 翻译成中文就是:无穷乘积 p_1 * p_2 * ... * p_n * ...收敛的定义是 p_i 中最多有
: 有限个是0,并且不是0的p_i构成的部分乘积收敛到一个有限的但不是0的数。
: 例如
: (1) 0 * 1 * 2 * ...* n *..不收敛,因为去掉0后的部分乘积不收敛。

o**a
发帖数: 76
13
我想我找到反例了
(1) \sum a_n 收敛,而 \prod (1+a_n) 不收敛
取 a_n = (-1)^(n+1) i/sqrt(n)
于是 a_n 是一个alternating series乘上i,容易看出\sum a_n收敛到某个有限的虚数
而\prod (1+a_n)不收敛,否则假设它收敛,则它的模也收敛
但是|1+a_n| = (1+1/n) 从而\prod_{n <= N} |1+a_n| = N+1 -> \infty 矛盾
(2) \prod (1+a_n) 收敛,而 \sum a_n不收敛
取 a_n = -1/n+i*(-1)^(n+1)*sqrt(2/n)
显然 \sum a_n的实部趋于负无穷,所以它不收敛
然而
(1+a_n)*(1+a_{n+1})
=1-2/sqrt(n+1)*[1/sqrt(n)-1/sqrt(n+1)]
+i*\sqrt(2)*(1+1/[sqrt(n)*sqrt(n+1)])*(1/sqrt(n)-1/sqrt(n+1))
容易看出上式右端去掉1后模被 1/n^{1.5}所控制,所以
\prod (1+a_n)收敛

t.

【在 o**a 的大作中提到】
: Ahlfors said that the convergence of \sum a_n is neither sufficient nor
: necessary condition for the convergence of \prod (1+a_n).
: (a_n are complex numbers)
: I've worked out the example that \prod (1+a_n) converges but \sum a_n doesn't.
: For example, a_n=-1/(n+1), \sum a_n -> -\infty, but \prod (1+a_n) -> 0
: Actually, as long as for all a_n, -1 < a_n < 0, then \prod (1+a_n) is positive
: and decreasing, so converges, but obviously \sum a_n may diverge.
: My question is, can you think of a series {a_n} s

o**a
发帖数: 76
14
事实上,第二个反例我是这样找出来的
由于\prod (1+a_n)收敛,容易看出一个必要条件是a_n->0
设a_n = x_n + i * y_n

log(1+a_n) = log |1+a_n| + i * arg(1+a_n)
= 0.5 * log (1+2*x_n+x_n^2+y_n^2) + i * arg(y_n/(1+x_n))
令x_n=-1/n,而y_n^2=2/n,则
log(1+a_n)实部变为0.5*log(1+1/n^2),部分和收敛
对于虚部,由于x_n->0, 所以符号决定于y_n
我们只要让y_n是alternating series,并且|y_n|->0,虚部就也是一个alternating
series,并且绝对值趋于0,所以部分和收敛

【在 o**a 的大作中提到】
: 我想我找到反例了
: (1) \sum a_n 收敛,而 \prod (1+a_n) 不收敛
: 取 a_n = (-1)^(n+1) i/sqrt(n)
: 于是 a_n 是一个alternating series乘上i,容易看出\sum a_n收敛到某个有限的虚数
: 而\prod (1+a_n)不收敛,否则假设它收敛,则它的模也收敛
: 但是|1+a_n| = (1+1/n) 从而\prod_{n <= N} |1+a_n| = N+1 -> \infty 矛盾
: (2) \prod (1+a_n) 收敛,而 \sum a_n不收敛
: 取 a_n = -1/n+i*(-1)^(n+1)*sqrt(2/n)
: 显然 \sum a_n的实部趋于负无穷,所以它不收敛
: 然而

x******g
发帖数: 318
15
第一个的反例不太好构造,实际上如果限制an为实数列的话,那个命题就是对的.只需
要利用x>-1时,ln(1+x)-x<=0,这样sum(ln(1+an))=sum(an)+sum(ln(1+an)-an)(这个
极限一定存在,要么为有限数,要么为负无穷大).
不过如果继续沿着这个思路走得更远一点的话,就可以得到一个反例.现在我们重新来
看为什么实数的时候没有反例,这是因为x-ln(1+x)(x很小时)约等于x^2/2,如果x为
实数,这个数是恒大于等于0的,求和的话极限一定是存在(可能为正无穷大),但如果不
限制x为实数的话,就没有这样的结论了.具体的构造如下
an=(-1)^n*i/ln(n+2) 2^2k<=n<2^(2k+1)
(-1)^n/ln(n+2) 2^(2k+1)<=n<2^(2k+2)
具体的验证并不难,我就不写了.



【在 x******g 的大作中提到】
: 你那个"反例"给得不好,因为极限是负无穷大也算是有极限的.
: 实际上log(1+an)约等于an,所以\sum a_n=-00基本上和\prod (1+a_n)=0差不多的,不能算
: 是反例.
: 两个反例
: 1.
: a_n=(-1)^n*(n+1)
: \sum a_n存在,\prod (1+a_n)不存在
: 2.
: a_n=-1 n=1
: 随意,但要保证\sum a_n不存在 n为其他值时

x******g
发帖数: 318
16
不好意思,我忘了这个定义了.

【在 o**a 的大作中提到】
: 事实上,第二个反例我是这样找出来的
: 由于\prod (1+a_n)收敛,容易看出一个必要条件是a_n->0
: 设a_n = x_n + i * y_n
: 则
: log(1+a_n) = log |1+a_n| + i * arg(1+a_n)
: = 0.5 * log (1+2*x_n+x_n^2+y_n^2) + i * arg(y_n/(1+x_n))
: 令x_n=-1/n,而y_n^2=2/n,则
: log(1+a_n)实部变为0.5*log(1+1/n^2),部分和收敛
: 对于虚部,由于x_n->0, 所以符号决定于y_n
: 我们只要让y_n是alternating series,并且|y_n|->0,虚部就也是一个alternating

x******g
发帖数: 318
17
如果发散于无穷大也算是收敛呢?

【在 o**a 的大作中提到】
: 我想我找到反例了
: (1) \sum a_n 收敛,而 \prod (1+a_n) 不收敛
: 取 a_n = (-1)^(n+1) i/sqrt(n)
: 于是 a_n 是一个alternating series乘上i,容易看出\sum a_n收敛到某个有限的虚数
: 而\prod (1+a_n)不收敛,否则假设它收敛,则它的模也收敛
: 但是|1+a_n| = (1+1/n) 从而\prod_{n <= N} |1+a_n| = N+1 -> \infty 矛盾
: (2) \prod (1+a_n) 收敛,而 \sum a_n不收敛
: 取 a_n = -1/n+i*(-1)^(n+1)*sqrt(2/n)
: 显然 \sum a_n的实部趋于负无穷,所以它不收敛
: 然而

o**a
发帖数: 76
18
第二个反例和我一开始给出的不同,因为\prod (1+a_n)不收敛于0,按照定义它是收敛的
但是\sum a_n的实部还是趋于负无穷,所以不收敛

【在 x******g 的大作中提到】
: 如果发散于无穷大也算是收敛呢?
o**a
发帖数: 76
19
我把它用latex敲出来了
http://www.anywhereenterprises.com:80/1/1/a?a=dF&p=pQZsOWrl1Y5iy7POAApQZOAAY5i

【在 o**a 的大作中提到】
: 我想我找到反例了
: (1) \sum a_n 收敛,而 \prod (1+a_n) 不收敛
: 取 a_n = (-1)^(n+1) i/sqrt(n)
: 于是 a_n 是一个alternating series乘上i,容易看出\sum a_n收敛到某个有限的虚数
: 而\prod (1+a_n)不收敛,否则假设它收敛,则它的模也收敛
: 但是|1+a_n| = (1+1/n) 从而\prod_{n <= N} |1+a_n| = N+1 -> \infty 矛盾
: (2) \prod (1+a_n) 收敛,而 \sum a_n不收敛
: 取 a_n = -1/n+i*(-1)^(n+1)*sqrt(2/n)
: 显然 \sum a_n的实部趋于负无穷,所以它不收敛
: 然而

o**a
发帖数: 76
20
这个发散于无穷大算不算收敛
确实有时候不好把握

【在 x******g 的大作中提到】
: 如果发散于无穷大也算是收敛呢?
相关主题
[求助]cos(a)+K*cos(b) 如何化简成一个乘积形式Parabolic equation收敛问题
O(log(n))分解只有两个质数因子的合数看看这个,即收敛又发散的调和级数
问一个基础的分布收敛问题,谢谢!一道概率题请教!
进入Mathematics版参与讨论
o**a
发帖数: 76
21
限制a_n为实数,还是可以构造反例
a_n =(-1)^{n+1}/sqrt(n)
则\sum a_n 收敛到有限数
但是
让n从1到2N求和
\sum log(1+a_n)
=log(1+1)+log(1-1/sqrt(2))+log(1+1/sqrt(3))+log(1-1/sqrt(4))+...
+log(1+1/sqrt(2N-1))+log(1-1/sqrt(2N))
=log(1-1/sqrt(1*2))+log(1-1/sqrt(3*4))+...+log(1-1/sqrt((2N-1)*2N))
由于当n充分大之后
log(1-1/sqrt(n*(n+1)) < log(1-1/(n+1)) < -1/(n+1)
所以
\sum log(1+a_n) < \sum -1/(n+1) 发散到负无穷
不过像你说的,如果发散到无穷也算收敛的话,这就不算
但是一般都不会把发散到无穷算为收敛的吧?特别是\sum a_n收敛到一个有限数

【在 x******g 的大作中提到】
: 第一个的反例不太好构造,实际上如果限制an为实数列的话,那个命题就是对的.只需
: 要利用x>-1时,ln(1+x)-x<=0,这样sum(ln(1+an))=sum(an)+sum(ln(1+an)-an)(这个
: 极限一定存在,要么为有限数,要么为负无穷大).
: 不过如果继续沿着这个思路走得更远一点的话,就可以得到一个反例.现在我们重新来
: 看为什么实数的时候没有反例,这是因为x-ln(1+x)(x很小时)约等于x^2/2,如果x为
: 实数,这个数是恒大于等于0的,求和的话极限一定是存在(可能为正无穷大),但如果不
: 限制x为实数的话,就没有这样的结论了.具体的构造如下
: an=(-1)^n*i/ln(n+2) 2^2k<=n<2^(2k+1)
: (-1)^n/ln(n+2) 2^(2k+1)<=n<2^(2k+2)
: 具体的验证并不难,我就不写了.

x******g
发帖数: 318
22
奇怪的时间

【在 o**a 的大作中提到】
: 限制a_n为实数,还是可以构造反例
: a_n =(-1)^{n+1}/sqrt(n)
: 则\sum a_n 收敛到有限数
: 但是
: 让n从1到2N求和
: \sum log(1+a_n)
: =log(1+1)+log(1-1/sqrt(2))+log(1+1/sqrt(3))+log(1-1/sqrt(4))+...
: +log(1+1/sqrt(2N-1))+log(1-1/sqrt(2N))
: =log(1-1/sqrt(1*2))+log(1-1/sqrt(3*4))+...+log(1-1/sqrt((2N-1)*2N))
: 由于当n充分大之后

o**a
发帖数: 76
23
我刚好看到以前别人给的一个反例,就顺手贴上来了 :-)

【在 x******g 的大作中提到】
: 奇怪的时间
x******g
发帖数: 318
24
我是说帖子的显示时间:)
而且如果把极限为无穷大定义作发散的话,实数情况的反例似乎并不难构造.
即使限制an是正项级数

【在 o**a 的大作中提到】
: 我刚好看到以前别人给的一个反例,就顺手贴上来了 :-)
o**a
发帖数: 76
25
正项是不可能的
因为正项\sum a_n收敛,则它绝对收敛
从而有个定理说\sum log(1+a_n)也绝对收敛
因为(1-e)*|z| < |log(1+z)| < (1+e)*|z|, 其中0
【在 x******g 的大作中提到】
: 我是说帖子的显示时间:)
: 而且如果把极限为无穷大定义作发散的话,实数情况的反例似乎并不难构造.
: 即使限制an是正项级数

1 (共1页)
进入Mathematics版参与讨论
相关主题
看看这个,即收敛又发散的调和级数再来Ahlfors书上一道习题
一道概率题请教!这个级数收敛么?
问一个简单的拓扑问题真心求教一道随机收敛的问题
求教个极限问题convergence 和 validation
what's the genus of cos(sqrt(z))?问个数学分析的证明题,在线急等
[合集] one problem about infinite products求教两个正项级数的极限证明
help! Matrix question (interveiw question)请教一个级数公式的证明
这个series 是convergent 还是divergent?[求助]cos(a)+K*cos(b) 如何化简成一个乘积形式
相关话题的讨论汇总
话题: prod话题: sum话题: 收敛话题: sqrt话题: log